6
$\begingroup$

Let $C$ be a category with finite hom-sets. Suppose that $X$ and $Y$ are objects in $C$ such that $C(Z,X)\cong C(Z,Y)$ for any Z (with no naturality condition). For which categories $C$ does it follow that $X \cong Y$? (Of course, it is true for posets).

A somewhat related question is the following.

Let $C$ be a symmetric monoidal closed category. Suppose that $X$ and $Y$ are objects in $C$ such that $[X,Z]\cong [Y,Z]$ for any Z (with no naturality condition). For which categories $C$ does it follow that $X \cong Y$?

$\endgroup$
3
  • 2
    $\begingroup$ mathoverflow.net/a/269745 seems to gesture towards an (unpublished) answer $\endgroup$ Sep 29, 2020 at 20:21
  • 3
    $\begingroup$ after a bit of digging, I discovered that a sufficient condition based on similar factorization properties is proven in A. Pultr: Isomorphism types of objects in categories determined by numbers of morphisms. Acta Sci. Math. Szeged35(1973), 155–160 $\endgroup$ Sep 29, 2020 at 20:32
  • $\begingroup$ @martti : thanks for the very useful references. In fact I had a similar idea of proof based on factorization. So it seems that it is true for instance for $C = Set_f^G$ for any finite category $G$ (as well as for their dual). $\endgroup$ Sep 30, 2020 at 18:13

1 Answer 1

4
$\begingroup$

The first question is the main topic of two recent papers:

As far as I understand, the theorems give orthogonal sufficient conditions: it's unclear whether there is a general theorem subsuming both.

I don't believe there are answers for enriched categories yet.

$\endgroup$
3
  • $\begingroup$ One should also mention Lovàsz's hom-counting theorem by inclusion-exclusion principle by Shoma Fujino and Makoto Matsumoto (arxiv.org/abs/2206.01994). $\endgroup$ Jul 13, 2022 at 20:08
  • $\begingroup$ And Locally-finite extensive categories, their semi-rings, and decomposition to connected objects by the same authors (arxiv.org/abs/2207.05702). $\endgroup$ Jul 13, 2022 at 20:09
  • 1
    $\begingroup$ @IvanDiLiberti: thanks, I forgot about that paper – I've updated my answer to include it. $\endgroup$
    – varkor
    Jul 13, 2022 at 21:07

Your Answer

By clicking “Post Your Answer”, you agree to our terms of service and acknowledge you have read our privacy policy.

Not the answer you're looking for? Browse other questions tagged or ask your own question.